0
$\begingroup$

Let $\mathcal{E}(\mathbb{R})$ be the space of all $C^\infty$ functions on $\mathbb{R}$ with its usual topology, and $\mathcal{E}'(\mathbb{R})$ be the dual space with the weak* topology.

Let $(T_i)_{i\in I}$ be a net in $\mathcal{E}'(\mathbb{R})$ that converges to $T$ in $\mathcal{E}'(\mathbb{R})$.

Is $(T_i)_{i\in I}$ bounded (in the Topological Vector Space sense of absorption by dilations of neighbourhoods of $0$)?

$\endgroup$
1
  • 5
    $\begingroup$ This is not even true in the topological vector space $\mathbb{R}$. So you could find a counterexample where all the $T_i$ were constant multiples of the same distribution. Nets are not like sequences. $\endgroup$ Sep 20, 2018 at 20:15

1 Answer 1

3
$\begingroup$

Not in general. For example let $I_0$ be an arbitrary set and $(T_i)_{i\in I_0}\ \ $ be an arbitrary family of elements of $\mathcal E'$. Let $I=I_0\cup\{\alpha\}\ \ $ for some new element $\alpha$ and define a partial order on $I$ by setting $\alpha>i$ for any $i\in I_0$. Then $I$ is a directed set. Define a net by extending the given family by $T_\alpha=0$. This net converges to zero, actually, it is eventually stationary. But as the family $(T_i)_{i\in I_0}\ \ $ is arbitrary, it need not be bounded.

$\endgroup$

Your Answer

By clicking “Post Your Answer”, you agree to our terms of service and acknowledge you have read our privacy policy.

Not the answer you're looking for? Browse other questions tagged or ask your own question.